Difference between revisions of "2021 AMC 10B Problems/Problem 7"

(Created page with "no one knows yet")
 
(Video Solution by TheBeautyofMath)
(10 intermediate revisions by 6 users not shown)
Line 1: Line 1:
no
+
==Problem==
one
+
In a plane, four circles with radii <math>1,3,5,</math> and <math>7</math> are tangent to line <math>l</math> at the same point <math>A,</math> but they may be on either side of <math>l</math>. Region <math>S</math> consists of all the points that lie inside exactly one of the four circles. What is the maximum possible area of region <math>S</math>?
knows
+
 
yet
+
<math>\textbf{(A) }24\pi \qquad \textbf{(B) }32\pi \qquad \textbf{(C) }64\pi \qquad \textbf{(D) }65\pi \qquad \textbf{(E) }84\pi</math>
 +
==Solution==
 +
<asy>
 +
/* diagram made by samrocksnature */
 +
pair A=(10,0);
 +
pair B=(-10,0);
 +
draw(A--B);
 +
draw(circle((0,-1),1));
 +
draw(circle((0,-3),3));
 +
draw(circle((0,-5),5));
 +
draw(circle((0,7),7));
 +
dot((0,7));
 +
draw((0,7)--(0,0));
 +
label("$7$",(0,3.5),E);
 +
label("$l$",(-9,0),S);
 +
</asy>
 +
After a bit of wishful thinking and inspection, we find that the above configuration maximizes our area, which is <math>49 \pi + (25-9) \pi=65 \pi \rightarrow \boxed{\textbf{(D)}}</math>
 +
 
 +
~ samrocksnature
 +
 
 +
== Video Solution by OmegaLearn (Area of Circles and Logic) ==
 +
https://youtu.be/yPIFmrJvUxM
 +
 
 +
~ pi_is_3.14
 +
 
 +
==Video Solution by TheBeautyofMath==
 +
https://youtu.be/GYpAm8v1h-U?t=206
 +
 
 +
~IceMatrix
 +
==Video Solution by Interstigation==
 +
https://youtu.be/DvpN56Ob6Zw?t=555
 +
 
 +
~Interstigation
 +
 
 +
{{AMC10 box|year=2021|ab=B|num-b=6|num-a=8}}

Revision as of 01:07, 23 February 2021

Problem

In a plane, four circles with radii $1,3,5,$ and $7$ are tangent to line $l$ at the same point $A,$ but they may be on either side of $l$. Region $S$ consists of all the points that lie inside exactly one of the four circles. What is the maximum possible area of region $S$?

$\textbf{(A) }24\pi \qquad \textbf{(B) }32\pi \qquad \textbf{(C) }64\pi \qquad \textbf{(D) }65\pi \qquad \textbf{(E) }84\pi$

Solution

[asy] /* diagram made by samrocksnature */ pair A=(10,0); pair B=(-10,0); draw(A--B); draw(circle((0,-1),1)); draw(circle((0,-3),3)); draw(circle((0,-5),5)); draw(circle((0,7),7)); dot((0,7)); draw((0,7)--(0,0)); label("$7$",(0,3.5),E); label("$l$",(-9,0),S); [/asy] After a bit of wishful thinking and inspection, we find that the above configuration maximizes our area, which is $49 \pi + (25-9) \pi=65 \pi \rightarrow \boxed{\textbf{(D)}}$

~ samrocksnature

Video Solution by OmegaLearn (Area of Circles and Logic)

https://youtu.be/yPIFmrJvUxM

~ pi_is_3.14

Video Solution by TheBeautyofMath

https://youtu.be/GYpAm8v1h-U?t=206

~IceMatrix

Video Solution by Interstigation

https://youtu.be/DvpN56Ob6Zw?t=555

~Interstigation

2021 AMC 10B (ProblemsAnswer KeyResources)
Preceded by
Problem 6
Followed by
Problem 8
1 2 3 4 5 6 7 8 9 10 11 12 13 14 15 16 17 18 19 20 21 22 23 24 25
All AMC 10 Problems and Solutions